IQ scores are normally distributed with a mean of 100 and a standard deviation of 15. Out of a randomly selected 1450 people from the population, how many of them would have an IQ between 106 and 125, to the nearest whole number?

IQ Scores Are Normally Distributed With A Mean Of 100 And A Standard Deviation Of 15. Out Of A Randomly

Answers

Answer 1

Answer:

Step-by-step explanation:

To find out how many people would have an IQ between 106 and 125, we need to calculate the area under the normal distribution curve between these two IQ values. We can do this by calculating the z-scores corresponding to these IQ values and then using a standard normal distribution table or a calculator.

First, let's calculate the z-score for an IQ of 106 using the formula:

z = (x - μ) / σ

where x is the IQ score (106), μ is the mean (100), and σ is the standard deviation (15).

z = (106 - 100) / 15 = 0.4

Next, let's calculate the z-score for an IQ of 125:

z = (x - μ) / σz = (125 - 100) / 15 = 1.67

Using a standard normal distribution table or a calculator, we can find the corresponding cumulative probabilities for these z-scores.

The cumulative probability for a z-score of 0.4 is approximately 0.6554.The cumulative probability for a z-score of 1.67 is approximately 0.9525.

To find the proportion of people with an IQ between 106 and 125, we subtract the cumulative probability corresponding to the lower z-score from the cumulative probability corresponding to the higher z-score:

0.9525 - 0.6554 = 0.2971

This means that approximately 29.71% of the population falls within the IQ range of 106 to 125.

To find out how many people out of the randomly selected 1450 would have an IQ in this range, we multiply the proportion by the sample size:

0.2971 * 1450 ≈ 431.15

Rounding to the nearest whole number, we find that approximately 431 people would have an IQ between 106 and 125 out of the randomly selected sample of 1450 individuals.

Answer 2

Answer:

To solve this question, we'll need to use the properties of normal distribution. Specifically, we'll use the z-scores, which tell us how many standard deviations away from the mean a given value is. The formula for calculating a z-score is:

Z = (X - μ) / σ

where:

- X is the value we're interested in,

- μ is the mean, and

- σ is the standard deviation.

The z-score corresponding to an IQ of 106 is (106 - 100) / 15 ≈ 0.40, and the z-score corresponding to an IQ of 125 is (125 - 100) / 15 ≈ 1.67.

Next, we need to determine the proportion of individuals in a normal distribution that falls between these z-scores. This is found by looking up these z-scores in a standard normal distribution table, or using a software function that provides the cumulative distribution function of the standard normal distribution.

The approximate values are:

- The cumulative probability associated with a z-score of 0.40 is about 0.6554.

- The cumulative probability associated with a z-score of 1.67 is about 0.9525.

The proportion of individuals with IQs between 106 and 125 is the difference between these probabilities, or about 0.9525 - 0.6554 = 0.2971.

Now, to find the number of people out of 1450 with IQs between 106 and 125, we simply multiply this proportion by 1450 and round to the nearest whole number:

0.2971 * 1450 ≈ 431

So, we expect about 431 people out of the randomly selected 1450 to have an IQ between 106 and 125.


Related Questions

Two homebuyers are financing $137,000 to purchase a condominium. They obtained a 15-year, fixed-rate loan with a rate of 5.05%. They have been given the option of purchasing up to four points to lower their rate to 4.81%. How much will the four points cost them?

$1,370
$1,730
$4,580
$5,480

Answers

The cost of four points is:4 x $1,370 = $5,480Thus, the four points will cost the homebuyers $5,480.

Points can help lower mortgage rates on fixed-rate loans. The concept of points, which are basically prepaid interest, is a little complicated.

Each point is worth one percent of the loan amount, and paying points can lower your interest rate by a certain amount, typically about one-eighth to one-quarter of a percentage point.

The cost of points in the given scenario can be found using the following steps:

The loan amount to purchase a condominium is $137,000. The homebuyers obtained a 15-year fixed-rate loan with a rate of 5.05%.

If the homebuyers opt for four points, their loan rate will decrease to 4.81%.

To figure out how much the points will cost the homebuyers, we must first determine the cost of one point. Since one point is equal to 1% of the loan amount, one point on a $137,000 loan is:1% of $137,000 = $1,370

To learn more about : cost

https://brainly.com/question/2292799

#SPJ8

3) Last year the mean salary for professors in a particular community college was $62,000 with a standard deviation of $2000. A new two year contract is negotiated. In the first year of the contract, each professor receives a $1500 raise.

Find the mean and standard deviation for the first year of the contract.
b) In the second year of the contract, each professor receives a 3% raise based on their salary during the first year of the contract. Find the mean and the standard deviation for the second year of the contract.

Answers

a) Mean for the first year of the contract: $63,500

The standard deviation for the first year of the contract: $2,000.

b) Mean for the second year of the contract: $65,405.

The standard deviation for the second year of the contract: $60.

We have,

To find the mean and standard deviation for the first year of the contract, we can use the given information and the properties of the normal distribution.

Given:

The mean salary for professors in the previous year = $62,000

Standard deviation in the previous year = $2,000

Raise in the first year = $1,500

Mean for the first year of the contract:

The mean salary for the first year can be obtained by adding the raise to the previous mean:

Mean = Previous Mean + Raise

Mean = $62,000 + $1,500

Mean = $63,500

The standard deviation for the first year of the contract:

Since each professor receives the same raise, the standard deviation remains the same:

Standard Deviation = $2,000

Therefore, for the first year of the contract, the mean salary is $63,500, and the standard deviation remains $2,000.

Now,

In the second year of the contract, each professor receives a 3% raise based on their salary during the first year of the contract.

To find the mean and standard deviation for the second year, we can use the given information and the properties of the normal distribution.

Mean for the second year of the contract:

To calculate the mean for the second year, we need to add a 3% raise to the mean salary of the first year:

Mean = Mean of the first year + (3% * Mean of the first year)

Mean = $63,500 + (0.03 * $63,500)

Mean = $63,500 + $1,905

Mean = $65,405

The standard deviation for the second year of the contract:

Since each professor receives a raise based on their salary from the first year, the standard deviation also increases. To calculate the standard deviation, we multiply the standard deviation from the first year by the percentage increase:

Standard Deviation = Standard Deviation of the first year * (Percentage Increase / 100)

Standard Deviation = $2,000 * (3 / 100)

Standard Deviation = $2,000 * 0.03

Standard Deviation = $60

Therefore, for the second year of the contract, the mean salary is $65,405, and the standard deviation is $60.

Thus,

a) Mean for the first year of the contract: $63,500

The standard deviation for the first year of the contract: $2,000.

b) Mean for the second year of the contract: $65,405.

The standard deviation for the second year of the contract: $60.

Learn more about mean here:

https://brainly.com/question/23263573

#SPJ1

(q11) Find the center of mass of the system of objects that have masses 2 , 3 and 5 at the point (-1,2),(1,1) and (3,3) respectively.

Answers

The center of mass of the system is approximately (3.7, 2.6).

The center of mass of a system of objects is the point where all the weight of the system appears to be concentrated. It can be defined as the average location of the weighted parts of the system.

The center of mass of a system is dependent on the mass of the objects in the system and their positions.

Let's determine the center of mass of the system with masses of 2, 3, and 5 at the points (-1, 2), (1, 1), and (3, 3), respectively. Let's name the masses m1, m2, and m3, respectively, and the coordinates (x1, y1), (x2, y2), and (x3, y3).

The x-component of the center of mass is given by the formula:

x= (m1x1 + m2x2 + m3x3) / (m1 + m2 + m3)

The y-component of the center of mass is given by the formula:

y= (m1y1 + m2y2 + m3y3) / (m1 + m2 + m3)

By using the given values, let's calculate the x and y components of the center of mass:

x = (2 x -1 + 3 x 1 + 5 x 3) / (2 + 3 + 5) = 37/10 ≈ 3.7y

= (2 x 2 + 3 x 1 + 5 x 3) / (2 + 3 + 5)

= 26/10 = 2.6

To learn more about : mass

https://brainly.com/question/28916233

#SPJ8

The number of combinations of eight items taken three at a time can be written as

Answers

Answer: 8C3

Step-by-step explanation: You need to use Combinations for this. Out of 8, you need to select 3, so answer is 8C3.

Multiply three consecutive digits backwards starting from 8, and divide by 3 factorial

(8*7*6)/(3*2*1)

=56

a is an arithmetic sequence where the 1st term of the sequence is {\textstyle\frac{3}{2}} and the 13th term of the sequence is -{\textstyle\frac{81}{2}}. Find the 13th partial sum of the sequence.

Answers

Answer:

195

Step-by-step explanation:

a = 3/2

According to the formula tn= a + (n-1)d

81/2= 3/2 + (13 - 1)d

81/2= 3/2 + 12d

81/3 = 12d

Therefore 27/12 = d

Sn= n/2 [2a + (n-1)d]

[tex]S_{13}[/tex] = 13/2 [2(3/2) + (13-1)(27/12)]

     = 13/2 (3 + 27)

     = 39/2 + 351/2

     = 390/2

     = 195

Suppose there are 17 jelly beans in a box-2 red, 3 blue, 4 white, and 8 green. What part of the jelly beans is blue? As a decimal rounded to the nearest ten-thousandth (four decimal places)

Answers

Blue Jelly beans are 0.1764 part of total .

Given,

Total beans = 17

Blue = 3

Red =2

White =4

Green =8

Now,

Out of total , green jelly beans = 8/17

Out of total , red jelly beans = 2/17

Out of total , white jelly beans = 4/17

Out of total , blue jelly beans = 3/17

Hence the blue jelly beans are 0.1764 part of total jelly beans .

Know more about decimal,

https://brainly.com/question/8985071

#SPJ1

I NEED HELP WITH STATISTICS

Answers

Am here for you so need anything don’t message me

Determine the a) total annual cost, and b) cost per mile to the nearest cent.
1. Liz Nolan drove 34,500 miles last year. The total of fixed costs was $9,916 and of variable costs was
$4,897.

Answers

Answer:

total annual cost: 49313

cost per mile: 14 cents

Step-by-step explanation:

find total annual cost by adding everything up

find cost per mile by doing 4897/34500

cost/ miles

we use variable cost since the only thing that might change each year is the amount of miles they drive

fixed costs are fixed and don't change

prove that the points 2, -1+i√3, -1-i√3 for a equilateral triangle on the argand plane.
Find the length of a side of this trangle?

Answers

Answer:

The lengths are equal so the triangle is equilateral

Step-by-step explanation:

We can write the points as follows,

(2,0), (-1,[tex]\sqrt{3}[/tex]) (-1,-[tex]\sqrt{3}[/tex])

now if it is an equilateral triangle, all side lengths must be equal

first we compute the sides(vectors)

(2-(-1),-[tex]\sqrt{3}[/tex]) = (3,-[tex]\sqrt{3}[/tex]) = side 1

(2-(-1),[tex]\sqrt{3}[/tex]) = (3,[tex]\sqrt{3}[/tex]) = side 2

(-1+1,[tex]\sqrt{3}[/tex]+[tex]\sqrt{3}[/tex]) = (0,2[tex]\sqrt{3}[/tex]) = side 3

now we compute the lengths of the sides using pythagoras theorem

(3)^2 + (-[tex]\sqrt{3}[/tex])^2 = (length of side 1)^2 = 9 + 3 = 12

similarly, (3)^2 + ([tex]\sqrt{3}[/tex])^2 = 12 = Length of side 2 squared

and,( 2[tex]\sqrt{3}[/tex])^2 = length of side 3 squared = 12

since the squares are equal, so the lengths must also be equal

so the triangle is equilateral

this is just a quick addition to the superb posting by "hamza0100" above

well, indeed, in the argand or imaginary plane, for those values above we have the coordinates of A(2 , 0) , B(-1 √3) and C(-1 , -√3), let' use the distance formula for those fellows

[tex]~\hfill \stackrel{\textit{\large distance between 2 points}}{d = \sqrt{( x_2- x_1)^2 + ( y_2- y_1)^2}}~\hfill~ \\\\[-0.35em] ~\dotfill\\\\ A(\stackrel{x_1}{2}~,~\stackrel{y_1}{0})\qquad B(\stackrel{x_2}{-1}~,~\stackrel{y_2}{\sqrt{3}}) ~\hfill AB=\sqrt{(~~ -1- 2~~)^2 + (~~ \sqrt{3}- 0~~)^2} \\\\\\ ~\hfill AB=\sqrt{( -3)^2 + ( \sqrt{3})^2} \implies \boxed{AB=\sqrt{ 12 }}[/tex]

[tex]B(\stackrel{x_1}{-1}~,~\stackrel{y_1}{\sqrt{3}})\qquad C(\stackrel{x_2}{-1}~,~\stackrel{y_2}{-\sqrt{3}}) \\\\\\ BC=\sqrt{(~~ -1- (-1)~~)^2 + (~~ -\sqrt{3}- \sqrt{3}~~)^2} \\\\\\ ~\hfill BC=\sqrt{( 0)^2 + ( -2\sqrt{3})^2} \implies \boxed{BC=\sqrt{ 12 }}[/tex]

[tex]C(\stackrel{x_1}{-1}~,~\stackrel{y_1}{-\sqrt{3}})\qquad A(\stackrel{x_2}{2}~,~\stackrel{y_2}{0}) ~\hfill CA=\sqrt{(~~ 2- (-1)~~)^2 + (~~ 0- (-\sqrt{3})~~)^2} \\\\\\ ~\hfill CA=\sqrt{( 3)^2 + (-\sqrt{3})^2} \implies \boxed{CA=\sqrt{ 12 }} \\\\[-0.35em] ~\dotfill\\\\ AB=BC=CA=\sqrt{12}\implies 2\sqrt{3}\hspace{5em}\qquad equilateral\textit{\LARGE \checkmark}[/tex]

A number divided by 10 is less than 4

Answers

2 is the correct answer

Answer: 2

Step-by-step explanation: 10 divided by 5 equals 2

Express 75 as a product of its prime factors write the prime factors in ascending order and give your answer in index form

Answers

Step-by-step explanation:

75 = 3 x 5 x 5    in prime factorization

Answer:

Step-by-step explanation:

3x5x5

Lucas and Mike went on a holiday with the same amount of money.
Each day, Lucas spent $260 while Mike spent $180. At the end of their holiday,
Lucas had $240 left while Mike had $720 left. How many days were they on holiday?

Answers

Let's represent the number of days they were on holiday with the variable "d".

We know that Lucas spent $260 per day and ended up with $240 left. Therefore, he spent a total of:

$260d + $240 = $260d - $(-$240) = $260d - $(-240)

We can also represent the amount spent by Mike as:

$180d + $720 = $180d + $720 - $0

We know from the problem that both of them started with equal amounts of money, so we can set these two expressions equal to each other:

$260d - $(-240) = $180d + $720 - $0

Simplifying this expression, we get:

$260d + $240 = $180d + $720

Subtracting $180d from both sides gives:

$80d + $240 = $720

Subtracting $240 from both sides gives:

$80d = $480

Dividing by 80 gives:

d = 6

Therefore, Lucas and Mike were on holiday for 6 days.


QUESTION 1 1.1 1.2 1.4 Use the definition of the derivative (first principles) to determine f'(x) if f(x)=2x 1.3 Determine f'(x) from first principles if f(x)=9-x². Determine f'(x) from first principles if f(x)=-4x².​

Answers

Based on the functions given, it should be noted that the values will be 2, -2x and -8x.

How to calculate the value

Using the definition of the derivative, we have:

f'(x) = lim(h->0) [f(x + h) - f(x)] / h

= lim(h->0) [2(x + h) - 2x] / h

= lim(h->0) 2h / h

= lim(h->0) 2

= 2

Therefore, f'(x) = 2.

For f(x) = 9 - x²:

Using the definition of the derivative, we have:

f'(x) = lim(h->0) [f(x + h) - f(x)] / h

= lim(h->0) [9 - (x + h)² - (9 - x²)] / h

= lim(h->0) [9 - (x² + 2xh + h²) - 9 + x²] / h

= lim(h->0) [-2xh - h²] / h

= lim(h->0) (-2x - h)

= -2x

Therefore, f'(x) = -2x.

For f(x) = -4x²:

Using the definition of the derivative, we have:

f'(x) = lim(h->0) [f(x + h) - f(x)] / h

= lim(h->0) [-4(x + h)² - (-4x²)] / h

= lim(h->0) [-4(x² + 2xh + h²) + 4x²] / h

= lim(h->0) [-4x² - 8xh - 4h² + 4x²] / h

= lim(h->0) [-8xh - 4h²] / h

= lim(h->0) (-8x - 4h)

= -8x

Therefore, f'(x) = -8x.

Learn more about functions on

https://brainly.com/question/31878183

#SPJ1

Use the image to determine the direction and angle of rotation.

A. 90°clockwise rotation
B. 90°counterclockwise rotation
C. 180°clockwise rotation
D. 270°counterclockwise rotation

Answers

C
180 is the correct answer.

Which is the equation of the given line in point-slope form?

y−0=−1(x−8)

y−0=1(x+8)

y=−x+8

y−8=−1(x+0)

Answers

Answer:

y = -x + 8

Step-by-step explanation:

Let's break down the equation step by step to understand it better.

The equation in point-slope form is given as:

y - y1 = m(x - x1)

In this case, we have:

y - 0 = -1(x - 8)

The point-slope form uses a specific point (x1, y1) on the line and the slope (m) of the line.

Here, the point (x1, y1) is (8, 0), which represents a point on the line. This means that when x = 8, y = 0. The graph has a point at (8, 0), which confirms this information.

The slope (m) is -1 in this equation. The slope represents the rate at which y changes with respect to x. In this case, since the slope is -1, it means that for every unit increase in x, y decreases by 1. The negative sign indicates that the line has a downward slope.

By substituting the values into the equation, we get:

y - 0 = -1(x - 8)

Simplifying further:

y = -x + 8

This is the final equation of the line in slope-intercept form. It tells us that y is equal to -x plus 8. In other words, the line decreases by 1 unit in the y-direction for every 1 unit increase in the x-direction, and it intersects the y-axis at the point (0, 8).

If the graph has points at (0, 8) and (8, 0), the equation y = -x + 8 accurately represents that line.

Find the measure of ∠F
.

Answers

Step-by-step explanation:

triangle EFG is an isosceles triangle

angle G

= 180°-58°

= 122° (adj. angles on a str. line)

angle F

= (180°-122°)÷2

= 29° (angles in a triangle)

I NEED HELP WITH STATISTICS

Answers

(a) The null hypothesis is that the mean birth weight of babies born at full term is 7.2 pounds. The alternative hypothesis is that the mean birth weight of babies born at full term is greater than 7.2 pounds.

(b) If the scientist decides to reject the null hypothesis, she might be making a Type I error.

(c) A Type II error occurs when the null hypothesis is false, but the scientist fails to reject it.

How to explain the information

a A Type I error occurs when the null hypothesis is true, but the scientist rejects it. In this case, the null hypothesis is that the mean birth weight of babies born at full term is 7.2 pounds. If the scientist rejects this hypothesis, she is saying that she believes that the mean birth weight is greater than 7.2 pounds. However, if the null hypothesis is true, then the mean birth weight is actually 7.2 pounds, and the scientist has made a mistake.

b In this case, the scientist would fail to reject the null hypothesis and conclude that the mean birth weight of babies born at full term is 7.2 pounds. However, the true mean birth weight is 7.7 pounds, so the scientist would be making a Type II error.

c In the context of a Type II error, suppose the null hypothesis is false, meaning there is indeed a significant difference or relationship. However, due to various factors such as insufficient sample size, low statistical power, or other limitations, the scientist fails to reject the null hypothesis. Consequently, they accept the null hypothesis even though it is false, leading to a Type II error.

Learn more about hypothesis on

https://brainly.com/question/606806

#SPJ1

546, 400 and 4,856 The value of 4 in which number is how many times larger than the value of 4 in which number.​

Answers

To determine how many times larger the value of 4 is in the second number compared to the first number, we need to calculate the ratio of the values.

First number: 546
Second number: 4,856

In the first number, the value of 4 is the same as the digit itself since it appears once.

In the second number, the value of 4 is larger since it appears twice.

To find the ratio, we divide the value of 4 in the second number by the value of 4 in the first number:

Value of 4 in second number: 2
Value of 4 in first number: 1

Ratio: 2/1 = 2

Therefore, the value of 4 in the second number is two times larger than the value of 4 in the first number.

Jessica needs to know how much water her new fish tank can hold:

A rectangular prism with a length of 8 inches, a width of 4 inches, and a height of 9 inches.

Determine the total volume of the fish tank.

Answers

The fish tank has a total volume of 288 inch³. As a result, Jessica's new fish tank has a capacity of 288 inch³ for water.

The volume of a rectangular prism can be calculated using the formula:

V = l x b x h..........(i)

where,

V ⇒ Volume

l  ⇒ length

b ⇒ width

h ⇒ height

From the question, we are given the values,

l = 8 inches

b = 4 inches

h = 9 inches

Putting these values in equation (i), we get,

V = 8 x 4 x 9

⇒ V = 288 in³

Therefore, the fish tank has a total volume of 288 inch³. As a result, Jessica's new fish tank has a capacity of 288 inch³ for water.

Learn more about the volume of rectangular prism on:

https://brainly.com/question/24284033

A scientist mixes water (containing no salt) with a solution that contains 35% salt. She wants to obtain 140 ounces of a mixture that is 15% salt. How many
ounces of water and how many ounces of the 35% salt solution should she use?

Answers

Answer:

.35x = 140(.15)

.35x = 21

x = 60 oz of 35% salt.

The scientist will need 60 oz of the 35% salt solution and 80 oz of water.

what is the greatest common factor of 97 and 24? what the answer

Answers

1

Because the number 97 is a prime number

Answer:

The greatest common factor (GCF) of two numbers is the largest number that divides evenly into both numbers. Since 97 is a prime number and 24 is not divisible by 97, the GCF of 97 and 24 is 1.

The amount of time a certain brand of light bulb lasts is normally distributed with a mean of 2000 hours and a standard deviation of 25 hours. Out of 665 freshly installed light bulbs in a new large building, how many would be expected to last between 2030 hours and 2060 hours, to the nearest whole number?

Answers

To determine the number of light bulbs expected to last between 2030 hours and 2060 hours, we need to calculate the z-scores corresponding to these values and then use the z-score formula to find the proportion of light bulbs within this range.

The z-score formula is given by:

z = (x - μ) / σ

where:

x = value

μ = mean

σ = standard deviation

For 2030 hours:

z1 = (2030 - 2000) / 25

For 2060 hours:

z2 = (2060 - 2000) / 25

Now, we can use the z-scores to find the proportions associated with each value using a standard normal distribution table or calculator. The table or calculator will provide the area/proportion under the normal curve between the mean and each z-score.

Let's calculate the z-scores and find the proportions:

z1 = (2030 - 2000) / 25 = 1.2

z2 = (2060 - 2000) / 25 = 2.4

Using a standard normal distribution table or calculator, we can find the proportions corresponding to these z-scores:

P(z < 1.2) ≈ 0.8849

P(z < 2.4) ≈ 0.9918

To find the proportion of light bulbs expected to last between 2030 hours and 2060 hours, we subtract the cumulative probabilities:

P(2030 < x < 2060) = P(z1 < z < z2) = P(z < z2) - P(z < z1)

P(2030 < x < 2060) ≈ 0.9918 - 0.8849

Finally, we multiply this proportion by the total number of light bulbs (665) to get the estimated number of light bulbs expected to last between 2030 hours and 2060 hours:

Number of light bulbs ≈ (0.9918 - 0.8849) * 665

Rounding to the nearest whole number, the expected number of light bulbs that would last between 2030 hours and 2060 hours is approximately 71.[tex]\huge{\mathfrak{\colorbox{black}{\textcolor{lime}{I\:hope\:this\:helps\:!\:\:}}}}[/tex]

♥️ [tex]\large{\textcolor{red}{\underline{\mathcal{SUMIT\:\:ROY\:\:(:\:\:}}}}[/tex]

plssssssssssssssssssssssssssssssssssssssssssss answe in 5 mins

Answers

Answer:

Because we are adding 2/5, we would be moving in the positive direction, which is to the right.

22% of what number is 3300

Answers

To find the number that corresponds to 22% of a given value, you can divide the given value by 22% (or 0.22).

Let's use this approach to find the number:

3300 ÷ 0.22 = 15,000

So, 22% of 15,000 is equal to 3300.

Answer:

x = 15000

Step-by-step explanation:

If you are using a calculator, simply enter 3300×100÷22, which will give you the answer.

vardan's homework assignment contains 24 problems of 58 1/3 of them are geometry. how many geometry problems are there?

Answers

There are 14 Geometry problems in Vardan's homework assignment.

The number of geometry problems in Vardan's homework assignment, we need to calculate 58 1/3 percent of the total number of problems.

First, let's convert 58 1/3 percent to a decimal by dividing it by 100:

58 1/3 percent = 58.33/100 = 0.5833

Next, we multiply the decimal by the total number of problems:

Number of geometry problems = 0.5833 * 24

To calculate this, we can multiply 0.5833 by 24:

Number of geometry problems = 0.5833 * 24 = 14

Therefore, there are 14 geometry problems in Vardan's homework assignment.

For more questions on Geometry .

https://brainly.com/question/31120908

#SPJ8

How do you solve the question Deloitte signs a contract on December 1 to provide 40 days of advisory services with receipt of $20,000 due at the end of the contract. On December 31, 75% of the services have been completed.

Answers

As of December 31, Deloitte should recognize $15,000 as revenue for the advisory services completed.

To solve the given question, we need to determine the amount of revenue that Deloitte should recognize as of December 31, based on the percentage of services completed.

Here's how we can calculate it:

Calculate the total revenue for the contract:

Total revenue = $20,000

Determine the percentage of services completed:

Percentage of services completed = 75%

Calculate the revenue recognized as of December 31:

Revenue recognized = Percentage of services completed × Total revenue

= 75% × $20,000

= $15,000

Therefore, as of December 31, Deloitte should recognize $15,000 as revenue for the advisory services completed.

Learn more about revenue click;

https://brainly.com/question/29567732

#SPJ1

Que número estoy pensando si al multiplicarlo por 4 y luego de sumarle 16 obtengo 8?

Answers

Answer:-2

Step-by-step explanation:

x(4)+16=8

Find the amplitude of this function.
In
++
t
Give your answer as a decimal.

Answers

Answer:

2.5

Step-by-step explanation:

The explanation is attached below.

Write a complete two-column proof for the following information.
Given: m1 = 62° and lines t and l intersect
Prove: m4 = 62°

Answers

The measure of angle 1 and angle 4 are equal, that is 62°, by vertical angle theorem.

Given that, m∠1=62° and lines t and l intersect.

From the given figure,

m∠1 and m∠4 are vertically opposite angles.

Vertical angles theorem or vertically opposite angles theorem states that two opposite vertical angles formed when two lines intersect each other are always equal (congruent) to each other.

Here, m∠1 =m∠4=62°

Hence, it is proved.

To learn more about the vertical angle theorem visit:

brainly.com/question/8119520.

#SPJ1

You spin the spinner once. 123 What is P(less than 2)? Write your answer as a fraction or whole number.

Answers

Answer:

See below

Step-by-step explanation:

Since the spinner has the numbers 1, 2, and 3 on it, and we want to find the probability of spinning a number less than 2, there is only one possible outcome that satisfies this condition, which is spinning a 1. Therefore, the probability of spinning a number less than 2 is:

P(less than 2) = P(1) = 1/3

So the probability of spinning a number less than 2 is 1/3.

Other Questions
Jane's utility function is represented as: U = F. C., F is quantity of food and C is quantity of clothing. If her budget constraint is represented as: 130 = 3F + 5C, her optimal bundle of consumption contains how many units of clothing? Explain why Wegner's use of shorelines to match continents in his reconstruction of Pangaea was criticized by other geoscientists and how that issue was subsequently resolved in Plate Tectonic Theory. 2- Explain briefly why a divergent plate boundary is also called a "constructive margin or boundary." 3- Briefly explain how density affects subduction. 4- Briefly explain how magnetic patterns on the seafloor support Plate Tectonic theory. 5- Briefly explain why the oceanic ridges are higher than the surrounding ocean basins. A particle moves along the curve x^2 = 4y. When x=2, thex-component of the velocity is changing at 3 mm/s. Find thecorresponding rate of change of the y-component of the velocity inmm/sec. Your company, based in Boston, Massachusetts, offers job interview training for people in the United States, Canada, England, and Australia.1. A brief description of your company here.2. Decide on some information that you can provide for your readers here. Create a headline and write the text. Perhaps you can write a commentary on the video or graphic that you choose to provide to the right of this text. Feel free to change the font size and color as you see fit, in any of the sectiions on this page. Also, please feel free to add sections to this page. If you right click on the page, you'll see options for adding rows and so forth.3. Information regarding when your company's training sessions are offered, and instructions for how persons can sign up to learn job interviewing skills.4. Write your own set of job interview tips and include them here for your web audience. Earlier today, the US government announced that the current inflation numbers as of June 10th has raised to a 40 year record high of 8.6%. Although rising inflation has been a hot headline for the last 12 months; the numbers were much worse than public expectation.These numbers have huge implication not just on Corporate America but also on the average citizen even here in Canada. As we have seen, gas prices have been surging for months on end without any relief. Consumer prices have also surged by 8.6% since 12 months earlier, leading to the cost of food, gas and other crucial household necessities to become almost unaffordable to the masses. Furthermore, with Rate hikes expected to continue, the stock market is not expected to see any relief in the short term and is in a full blow bear market; with a recession expected by many corporate giants.In my opinion, I dont believe that even these rate hikes will lower current inflation. With a 8.6% inflation rate, I believe that in order to even come close to getting the rate down to 2%, it will take much more than raising hikes for just a few months. With midterms coming up, I am unsure if the American government is willing to take a hit in GDP over inflation numbers, but without drastic and more sustained measures for at least until the remainder of the year; the inflation rate of 2% would likely just remain a pipe dream unless numbers are altered for cosmetic purposes just for the midterm. Like many others, I believe that there is a lot of corruption behind the scenes in terms of economic manipulation and control with the sentiment on many global leaders also at an all time low.What do you guys think of the current economic situation? Do you place the blame on the leaders or do you think this is just a natural cycle and regular course of the market? In your own words, describe the three specific interpersonal skills that you as a leader believe to be the MOST important to develop in order to successfully manage/lead teams to high performance in a diverse and changing cross-cultural work environment.(1) List & describe the three (3) skills you believe to be the most important,(2) Explain why each is necessary and(3) What you (or an individual) can do to acquire and sharpen each of these skills.Respond to the discussion board posts and comment on the opinions of at least two (2) other students within this module. Assume that you use the constant dividend growth model to value a stock. Which of the following will cause you to increase your valuation of the stock? a decreasing the required rate of return for the stock b. decreasing the estimated amount of next year's dividend e decreasing the expected dividend growth rate d. an announcement that the CFO has been fired. Complete an analysis of the key internal factors that have implications for successful implementation of your organizations strategy and goals/objectives(Morgan Stanley). Submit your work in your assignment folder in the form of an approximate 2,000-word double-spaced APA-formatted paper. The title page, reference list, and any appendices are not included in this suggested word count. You do not need to include an abstract.Your paper should address these topics:Given the companys Vision, Mission and Objectives (VMO), identify the companys core competencies and assess which ones are rare, costly, or not easily imitated. Discuss how they are related to and critical to the VMO execution.Present a summary of your organization's strengths and weaknesses. Submit the SWOT format in Table form and add in some narrative to discuss the strengths and weaknesses in more detail. Explain in your discussion (not in the table) why you selected them and how they relate to the VMO and organization strategy. (Note: You will have an opportunity to complete a full SWOT analysis, including threats and opportunities, as part of your week 6 paper.).Apply the Resource-Based View (RBV) to help you identify both the tangible and intangible assets your organization may be able to use to accomplish its intended strategies. You can list them in a table form and then follow with a discussion of the assets, why you selected them and how they relate to the VMO and strategy.Consider and discuss the things that may make your organization's resources and capabilities difficult for others to imitate. Use Value Chain Analysis to help you deepen your understanding of the relative value of the resources and capabilities you have identified. Seek objective and independently verifiable evidence of potential rarity of the resources and capabilities.IMPORTANT: Do not just use someone else's SWOT or other analysis. We want you to think for yourself. Critically analyze your firm and write about your original conclusions. Imagine you have been asked by the organizations CEO or top leader to offer an assessment of the organization and how well it is positioned (or not) to deliver on the VMO and strategy. This is a critical element, stand back and offer thoughtful criticism and recommendations.Add in a strong conclusion that ensures the reader leaves your paper with a clear recap of your key points. Simon Sinek believes that a good leader inspires people to accept the final results of a project without entertaining questions about why the project is worthy of pursuing. This enables great leaders to save time and money.TrueFalse If the price of blueberries rises, the quantity of strawberries consumed will decrease and the price of blueberry muffins will fall. Is this statement true or false?The rise in the price of blueberries ___ strawberries and ___ false; increases the demand for; increases the quantity of strawberries supplied false; increases the demand for; decreases the quantity of strawberries supplied false; decreases the demand for; increases the quantity of strawberries demanded true; decreases the demand for; decreases the quantity of strawberries supplied true; decreases the demand for; decreases the quantity of strawberries demanded Assume that the oil extraction company needs to extract Q units of oil (a depletable resource) reserve in a dynamically efficient manner. What should be a minimum amount of Q so that the oil reserve extraction can last for at least 13 periods if (a) the marginal willingness to pay for oil in each period is given by P = 39 -0.2q, (b) marginal cost of extraction is constant at $2 per unit, and (c) discount rate is 2%? Mutual fund investors delegate all of the following decisions to the fund's managers EXCEPT: A. when to buy and sell individual stocks. B. how much money to invest in the fund. C. how many securities to hold in the portfolio. D. which companies and industries to invest in. Donald received the following annual returns from his investment: Year 1 5.0% Year 2 -9.7% Year 3 14.2% Year 4 8.9% Calculate the Standard deviation of returns. Round the answer to two decimals places. The planning stage of integrating a vertical acquisition is difficult because: a. Vertical acquisitions are typically a rare occurrence for a firm b. Corporate staff are not interested in vertical acquisitions c Technological substitutes can raise the value of the acquisition d. Other acquirers may still want to buy the target In integration planning for a vertical acquisition, including the internal unit that receives the target's inputs is important because a. The unit's growth rate is higher than other businesses in the acquirer b. The unit needs to grow internationally c. The benefits to the unit from increased control are a major reason for the acquisition d. The acquirer is going through a major overhaul of its business portfolio The acquirer may be able to improve the target's performance in a vertical acquisition because a. The acquirer has specific resources or capabilities that may contribute to the target's external market position b. The acquirer's headquarters is based in Dallas c. The target is based in Dallas d. The target brings special resources and capabilities to the acquirerEquity markets value the merger of two firms most when: a. Their technologies and types of customer are very similar b. They are based on different continents c. One was founded many years before the other d. One is large and the other is small Which of the following is generally true for horizontal acquisitions? a. They take place in Manhattan b. They can be contentious because other bidders may be interested in the target c. Underpricing is common d. The acquirer and target have little knowledge of each other On January 1, 2021, Tiny Tim Industries had outstanding $1,000,000 of 11% bonds with a book value of $969,000. The indenture specified a call price of $985,500. The bonds were issued previously at a p Shoprite Group has created a new digital business unit called ShopriteX to use data science to "enhance customer experiences". One of its first projects is an artificial intelligence-powered store with no checkout counters. "Incubated over the past year, ShopriteX is combining data science and technology to create more personalised shopping experiences for customers," the JSE-listed retail giant said on Wednesday. The group said ShopriteX, which was responsible for the Xtra Savings rewards programme and the Checkers Sixty60 on-demand shopping app, is now piloting Checkers Rush, an automated, cashless "no queues, no checkout, no waiting" concept store in Brackenfell in Cape Town. The concept store allows Shoprite employees to "grab products and walk out". Using camera technology coupled with artificial intelligence software to identify the products being taken off the shelves, Checkers Rush bills users bank cards upon exit. Its not dissimilar to whats offered by Amazon.com through its Amazon Go retail stores in the US and the UK. "We are serious about being Africas most customer-centric retailer, and the launch of ShopriteX represents our investment in fit-for-the-future precision retail, which is increasingly digital and dataled," said CEO Pieter Engelbrecht in a statement. "The launch is part of the groups strategy to grow its ecosystem of value for consumers and monetise new and diverse revenue streams," he added. The 250-strong ShopriteX division includes data science, e-commerce and personalisation experts who work alongside the groups IT team to "create and implement new innovations". Shoprite has about a thousand employees working in its IT department. "The next era of growth for us is about precision retailing. ShopriteX will use our rich customer data to supercharge a smarter Shoprite and ultimately fuse the best of digital with our operational strength across the continent," said Engelbrecht. Source: TechCentral. 2021. Shoprite is piloting an Al-powered store with no checkouts. TechCentral, 18 Augustus 2022 questions As a marketing consultant you are tasked to compile marketing proposal of 750 900 words for the new business, ShopriteX of the Shoprite Group. Include the following in your report: Introduction Body a) Discuss the marketing orientation you prose ShopriteX should follow and provide reasons for your proposal. b) Outline the individual and group factors that will determine the consumer behaviour relevant to ShopriteX. Provide examples specific to ShopriteX. c) Compile a market segmentation for ShopriteX. d) Determine how the ShopriteX customer base should be targeted according to the segmentation your compiled. e) Explain which market position ShopriteX should take considering your recommended segmentation and targeting. Conclusion You are required to conduct additional research and use at least 2 source excluding your prescribed textbook, to compile this JTM's can its $150M in bonds - maturing in 7 years and paying a fixed 4.45% - for the same amount paying a floating rate of the Bloomberg Short Term Bank Yield Index + 1.50%. You are asked to show the cash flows for the fixed and floating scenarios and the net difference each year plus the net overall difference undiscounted and discounted using a 5% discount rate. Show all fixed and floating payments as negative cash flows. Net benefits use the formula: floating payments minus fixed payments. QUESTION 13 If a company's earnings before taxes (EBT) is 5000, and has a tax rate of 15%, while interest payments are 750 what is the company's net income? 3612.5 O 4887.5 O 4250 O None of the above Answer the following: Each sample of water has a(A + 10)% chance of containing a particular organic pollutant. Assume that the samples are independent with regard to the presence of the pollutant. Find the probability that in the next (A +18) samples a. Exactly 2 contains the pollutant. b. At least 4 contains the pollutant. c. Find mean and standard deviation of the samples Suppose you plan to buy a new house. The cost of the property is $350,000, and you need to take out a 30-year mortgage with monthly end-of-month payments. The lender requires a 20% down payment. The annual interest rate on your mortgage is 4.25%. Create an amortization schedule and answer the following question.The total interest you would pay in 30 years is $ _____. Round the result to the nearest integer.